ChaseDream
搜索
返回列表 发新帖
00:00:00

Economist: On average, the emergency treatment for an elderly person for injuries resulting from a fall costs $11,000. A new therapeutic program can significantly reduce an elderly person's chances of falling. Though obviously desirable for many reasons, this treatment program will cost $12,500 and thus cannot be justified.

Which of the following, if true, most seriously undermines the conclusion of the argument?

正确答案: C

更多相关帖子

524

帖子

15

好友

4712

积分

ChaseDream

注册时间
2003-03-17
精华
8
解析
查看: 1691|回复: 2
打印 上一主题 下一主题

Prep一道题不明白

[复制链接]
楼主
发表于 2008-7-25 17:49:00 | 只看该作者

Prep一道题不明白

Economist:  On average, the emergency treatment for an elderly person for injuries resulting from a fall costs $11,000.  A new therapeutic program can significantly reduce an elderly person's chances of falling.  Though obviously desirable for many reasons, this treatment program will cost $12,500 and thus cannot be justified.

 

Which of the following, if true, most seriously undermines the conclusion of the argument?

 

(A) Among elderly people who had followed the program for only a few months, the number of serious falls reported was higher than it was for people who had followed the program for its recommended minimum length of one year.

(B) Falls resulting in serious injuries are less common among elderly people living in nursing homes than they are among elderly people who live alone at home.

(C) A frequent result of injuries sustained in falls is long-term pain, medication for which is not counted among the average per-person costs of emergency treatment for elderly people's injuries from such falls.

(D) The new therapeutic program focuses on therapies other than medication, since overmedication can cause disorientation and hence increase the likelihood that an elderly person will have a serious fall.

(E) A significant portion of the cost of the new therapeutic program is represented by regular visits by health care professionals, the costs of which tend to increase more rapidly than do those of other elements of the program.

请问为什么选 C 呀          谢谢

沙发
发表于 2008-7-25 19:13:00 | 只看该作者

原文说老人跌倒costs $11,000, treatment program will cost $12,500 and thus cannot be justified 就是说由于预防的成本比治疗成本高所以不能justify

C说fall会造成长期病痛 而治疗这个长期病痛的cost是不包含在题干说的那个cost里的 所以实际由fall造成的cost远不止 $11,000 削弱推论

板凳
 楼主| 发表于 2008-7-26 19:43:00 | 只看该作者
谢谢
您需要登录后才可以回帖 登录 | 立即注册

Mark一下! 看一下! 顶楼主! 感谢分享! 快速回复:

手机版|ChaseDream|GMT+8, 2024-9-30 00:23
京公网安备11010202008513号 京ICP证101109号 京ICP备12012021号

ChaseDream 论坛

© 2003-2023 ChaseDream.com. All Rights Reserved.

返回顶部